Difference between revisions of "2021 JMPSC Accuracy Problems/Problem 9"

(Created page with "==Problem== If <math>x_1,x_2,\ldots,x_{10}</math> is a strictly increasing sequence of positive integers that satisfies <cmath>\frac{1}{2}<\frac{2}{x_1}<\frac{3}{x_2}< \cdots...")
(No difference)

Revision as of 21:29, 10 July 2021

Problem

If $x_1,x_2,\ldots,x_{10}$ is a strictly increasing sequence of positive integers that satisfies \[\frac{1}{2}<\frac{2}{x_1}<\frac{3}{x_2}< \cdots < \frac{11}{x_{10}},\] find $x_1+x_2+\cdots+x_{10}$.

Solution

asdf